Esse problema caiu na Olimpíada Iberoamericana de 2009 que eu participei.
Foi o problema 5 da prova e lá pedia para provar injetividade e
sobrejetividade.
Em qua, 17 de fev de 2021 00:16, Anderson Torres <
torres.anderson...@gmail.com> escreveu:
> Em dom., 14 de fev. de 2021 às 17:20, Claudio Buf
Eu gostaria de saber da origem desse problema...
Em dom., 14 de fev. de 2021 às 14:32, joao pedro b menezes <
joaopedrobmene...@gmail.com> escreveu:
> Obrigado pela resposta, mas ainda tenho umas dúvidas. Poderia dar um
> exemplo de tal função ou explicar como construí-la? E se f fosse somente
>
Em dom., 14 de fev. de 2021 às 17:20, Claudio Buffara
escreveu:
>
> Será que essa sequência é sobrejetiva (sobre os racionais positivos)?
> Porque como a(2^n) = n+1, ela certamente atinge todos os naturais, de modo
> que é ilimitada, superiormente e inferiormente (já que a(2^n + 1) = 1/(n+1) ).
>
Como a função x ---> 1/x3 , x > 0, é postiva e estritamente decrescente,
para todo inteiro positivo n temos que
Soma(1, n) 1/k^3 = 1 + Soma(2, n) 1/k^3 < 1 + Integral (2,n) 1/x^3 dx < 1
+ Integral (2, oo) 1/x^3 dx = 1 + [-1/(2x^2)] [2, oo) = 1 + 1/1/8 = 9/8 <
10/8 = 5/4
Em ter., 16 de fev. de 2
Será que essa sequência é sobrejetiva (sobre os racionais positivos)?
Porque como a(2^n) = n+1, ela certamente atinge todos os naturais, de modo que
é ilimitada, superiormente e inferiormente (já que a(2^n + 1) = 1/(n+1) ).
Mesmo que não seja, seria interessante descobrir que racionais positivos e
Obrigado pela resposta, mas ainda tenho umas dúvidas. Poderia dar um
exemplo de tal função ou explicar como construí-la? E se f fosse somente
injetora, mudaria alguma coisa?
--
Esta mensagem foi verificada pelo sistema de antiv�rus e
acredita-se estar livre de perigo.
Em dom., 14 de fev. de 2021 às 11:30, joao pedro b menezes <
joaopedrobmene...@gmail.com> escreveu:
> Obs: f é bijetora
>
>>
>
Acho que nao basta. Se f(x)=y entao f(x+y)=x+f(y).
Com isso, poderiamos fazer uma funcao que nao aja linearmente em (0,1) mas
aja linearmente fora dele.
> --
> Esta me
Em sáb., 13 de fev. de 2021 às 17:56, Jeferson Almir <
jefersonram...@gmail.com> escreveu:
> Amigos, peço ajuda em provar a injetividade dessa sequência que seria uma
> saída para provar a unica ocorrência do racional que aparece nela. Estou
> andando em círculos tentando montar uma possível induç
a(1) = 1
a(2n) = a(2n-1) + 1
a(2n+1) = 1/a(2n)
Fazendo a(n) = p(n)/q(n), obtemos duas sequências: p(n) e q(n).
E elas são tais que:
p(1) = q(1) = 1
p(2n) = p(2n-1) + q(2n-1)
q(2n) = q(2n-1)
p(2n+1) = q(2n)
q(2n+1) = p(2n)
Como as sequências começam com 1 e 1, que são primos entre si, e como
mdc(p
Ué! Continua sendo. Só que é outra questão...
On Sun, Feb 14, 2021 at 3:34 AM Ralph Costa Teixeira
wrote:
> Sim, voce tem razao -- eu achei que era a_2n = a_{2n-1} +1. Que pena, era
> uma boa questao com Fibonacci. :)
>
> On Sun, Feb 14, 2021 at 12:35 AM Claudio Buffara <
> claudio.buff...@gma
Sim, voce tem razao -- eu achei que era a_2n = a_{2n-1} +1. Que pena, era
uma boa questao com Fibonacci. :)
On Sun, Feb 14, 2021 at 12:35 AM Claudio Buffara
wrote:
> Oi, Ralph:
>
> Eu posso ter entendido errado a definição da sequência, mas achei termos
> diferentes dos seus:
> 1: 1
> 2: 2
> 3
Oi, Ralph:
Eu posso ter entendido errado a definição da sequência, mas achei termos
diferentes dos seus:
1: 1
2: 2
3: 1/2
4: 3
5: 1/3
6: 3/2
7: 2/3
8: 4
9: 1/4
10: 4/3
11: 3/4
12: 5/2
13: 2/5
14: 5/3
15: 3/5
16: 5
...
[]s,
Claudio.
On Sat, Feb 13, 2021 at 7:59 PM Ralph Costa Tei
Se a sequência é:
a(1) = 1
a(2n) = a(n) + 1
a(2n+1) = 1/a(2n),
então:
Como os termos da sequência são positivos, os termos de ordem par são
maiores do que 1 e os de ordem ímpar (e maior do que 1) são menores do que
1.
Se houver alguma repetição, então o primeiro termo a(n) a ser repetido
deverá índ
Meio enrolado, vou escrever meio vagamente.
Eu sugiro olhar primeiro para os caras com indice impar. Sao eles:
a1=1/1
a3=1/2
a5=2/3
a7=3/5
a8=5/8
...
Ou seja, mostre que eles sao quocientes de numeros de Fibonacci
consecutivos (os caras de indice par sao os inversos desses). Agora tem
varias manei
Disfarce o Lema da Boa Ordenacao, dado que e equivalente ao principio da
inducao.
Em sex., 5 de fev. de 2021 às 07:31, joao pedro b menezes <
joaopedrobmene...@gmail.com> escreveu:
> obs: só agora fui ver o título :) , se era necessário fazer especialmente
> por indução, por favor desconsidere a
obs: só agora fui ver o título :) , se era necessário fazer especialmente
por indução, por favor desconsidere a minha resposta.
On Fri, Feb 5, 2021 at 7:14 AM joao pedro b menezes <
joaopedrobmene...@gmail.com> wrote:
> Suponha que d | (a^(2)^n ) + 1. Então a^2^n = -1 (mod d). Pegue um primo
> ta
Suponha que d | (a^(2)^n ) + 1. Então a^2^n = -1 (mod d). Pegue um primo
tal que p| d, então a^2^n = -1 (mod p). Mas temos: a^2^(n+1) = 1 (mod p).
Logo
ord(p)a | 2^(n+1), mas ord(p)a não divide 2^n, logo ord(p)a = 2^(n + 1).
Isso é um absurdo, pois ord(p)a < p <= d <= 2^(n + 1).
obs: tenho quase c
Hm, confere o enunciado - era parte inteira, ou inteiro mais proximo?
On Wed, Feb 3, 2021, 18:39 joao pedro b menezes
wrote:
> Obrigado pela dica! Honestamente creio que existe um erro nesse problema.
> Fazendo alguns casos na mão é possivel perceber que isso sempre resulta em
> 8n + 7. Essa é a
Obrigado pela dica! Honestamente creio que existe um erro nesse problema.
Fazendo alguns casos na mão é possivel perceber que isso sempre resulta em
8n + 7. Essa é a prova:
"Provar que ( n^(1/3) + ( n + 2)^(1/3) )³ < 8n + 8. Abrindo a potência,
temos:
2n + 2 + 3 * ( (n² ( n + 2))^(1/3) + (n(n + 2)
Sem tempo agora, mas olhando por alto eu aproximaria o que estah dentro do
() por 2(n+1)^(1/3), o que levaria imediatamente a 8(n+1). Serah que a
parte inteira daquela coisa eh 8(n+1)?
Entao eu tentaria abrir os cubos, subtrair 8(n+1), e mostrar que o que
sobra eh menor que 1.
Serah que funciona?
Olá, boa noite, obrigado pela resposta( e pela dica)! Quanto ao meu
conhecimento de cálculo, embora saiba um pouco, ele é limitado e portanto
não conhecia esse teorema. Já li sobre ele durante o dia e entendi sua
demonstração. Mais uma vez, obrigado aos dois!
Em geral, sempre que você não sabe o que fazer com uma potência (por
exemplo nesse caso em que tanto a base quanto o expoente dependem de
x), a dica é trocar a base B por e^(log(B)).
Trocando (1+x) por e^(log(1+x)), vai ficar:
e^( ln(1+x) / x )
Como a função f(u)=e^u é contínua, basta saber
Oi, João Pedro. Voce sabe Calculo -- em particular, a Regra de L'Hopital?
Isso eh o que os livros chamam de "indeterminação do tipo Infinito^0". Ao
inves de achar o limite desta função, vamos passar o logaritmo primeiro,
lembrando que
ln (1+x)^(1/x) = 1/x * ln(1+x)
ou seja, ache primeiro este limi
Muito Obrigado!!! Me empolguei tanto com sua resolução que no final quase
apluadi de pé, nao o fiz pois quem mora comigo iria duvidar da minha
sanidade.
Em sex, 29 de jan de 2021 20:11, Claudio Buffara
escreveu:
> Ponha a = raiz(2).
> Então, vc precisa provar que, para n >= 2, a^(2n) > 1 + n*a^(
Ponha a = raiz(2).
Então, vc precisa provar que, para n >= 2, a^(2n) > 1 + n*a^(n-1) <==> a^n
> 1/a^n + n/a.
Pra n = 2 isso é verdade.
Suponha que, para um dado n >= 2, 1/a^n + n/a < a^n (H.I.)
Então 1/a^(n+1) + (n+1)/a < 1/a^n + 1/a + n/a = 1/a + (1/a^n + n/a) < 1/a +
a^n (pela H.I.)
Agora, rest
Olá, Ralph!
Sim, serve! Com certeza!
Muito obrigado!
Abraços!
Luiz
Em qui, 28 de jan de 2021 1:59 PM, Ralph Costa Teixeira
escreveu:
> A wikipedia tem um comecinho:
> https://pt.wikipedia.org/wiki/Desarranjo
> https://en.wikipedia.org/wiki/Derangement
> Serve?
>
> On Thu, Jan 28, 2021 at 1:15 PM
A wikipedia tem um comecinho:
https://pt.wikipedia.org/wiki/Desarranjo
https://en.wikipedia.org/wiki/Derangement
Serve?
On Thu, Jan 28, 2021 at 1:15 PM Luiz Antonio Rodrigues <
rodrigue...@gmail.com> wrote:
> Olá, pessoal!
> Boa tarde!
> Estou acompanhando com interesse a discussão, mas gostaria
Ok, vamos escrever a primeira linha como:
a= tb
c=(-1-t)d
A segunda linha diz que t^2.b^2+(1+t)^2.d^2=1, ou seja,
t^2 + 2t.d^2 + d^2 = 1 (**)
(Estou tentando botar tudo em termos de t e d!)
Agora: b^3/a + d^3/c = b^2/t - d^2/(1+t) = (1-d^2)/t - d^2/(1+t) =
= (1-2t.d^2 +t -d^2) / (t^2+t)
Use (**
2^2+2^2=2^2+2^2 --- serve?
De: owner-ob...@mat.puc-rio.br em nome de Caio
Costa
Enviado: domingo, 24 de janeiro de 2021 12:35
Para: obm-l@mat.puc-rio.br
Assunto: [obm-l] x^a + y^b = a^x + b^y
Prove ou disprove que a equação acima não tem solução para x, y, a, b
Olá, pessoal!
Boa tarde!
Estou acompanhando com interesse a discussão, mas gostaria de pedir uma
indicação de site ou outro material que trate de permutações caóticas.
Muito obrigado!
Abraços!
Luiz
Em qui, 28 de jan de 2021 11:38 AM, Arthur Queiroz
escreveu:
> Uma pergunta: você assume que o núm
Souberam que a questão foi realmente anulada?
https://g1.globo.com/educacao/enem/2020/noticia/2021/01/27/inep-anula-duas-questoes-do-enem-2020.ghtml
Em qui., 28 de jan. de 2021 às 11:38, Arthur Queiroz
escreveu:
> Uma pergunta: você assume que o número de sorteios é !10. Mas e se, em
> meio ao s
Muito obrigado a todos pelas mensagens.
Como a gente aprende por aqui!!!
No fim das contas a questão foi anulada pelo INEP.
Como disse o Claudio Buffara, daria um ótimo artigo!
Em qui., 28 de jan. de 2021 às 11:38, Arthur Queiroz
escreveu:
> Uma pergunta: você assume que o número de sorteios é
Uma pergunta: você assume que o número de sorteios é !10. Mas e se, em meio
ao sorteio, nossa permutação caótica seja tal que seja formado um ciclo
indesejado? Digamos A->B->C->A. Como o sorteio continuará nesse caso? Será
escolhida aleatoriamente uma pessoa de fora do ciclo para continuar? Isso
nã
Mas daí me parece que temos 3 conjuntos distintos (supondo que ninguém se
auto-presenteia):
1) o dos desarranjos de N pessoas;
2) o das sequências de N presenteados;
3) o dos diferentes jogos de amigo oculto com N pessoas (que o seu exemplo
mostrou ser diferente de (2): duas sequências idênticas de
Muito obrigado, Ralph!
Muito interessante!
Meu caso particular foi pequeno demais.
Daí eu só vi a situação em que um dado desarranjo origina duas (ou mais)
sequências distintas de presenteados.
Mas, como vc bem mostrou, com 6 ou mais participantes pode ocorrer a
situação "dual": uma mesma sequênc
De: owner-ob...@mat.puc-rio.br em nome de Ralph
Costa Teixeira
Enviado: quarta-feira, 27 de janeiro de 2021 01:37
Para: obm-l@mat.puc-rio.br
Assunto: Re: [obm-l] Amigo secreto ENEM
Fiz uma versão ligeiramente mais "limpa" do que escrevi antes, vejam se vocês
gostam mais:
1
Fiz uma versão ligeiramente mais "limpa" do que escrevi antes, vejam se
vocês gostam mais:
1) COM AUTO-SORTEIOS:
p(Mesma Pessoa Inicia e Termina) = p(Apenas um Grande Ciclo de Tamanho N) =
(N-1)! / N!=1/N
Portanto, p(Pessoas diferentes Iniciam e Terminam) = 1-1/N
Por simetria esta segunda probabil
Oi, Claudio.
Primeiro, parece que o video supõe que NÃO podem haver "auto-sorteios"
(isto fica implícito quando ele diz que a primeira a entregar não pode ser
a primeira e receber nem a penúltima, evitando que o último se de um
presente). Vou supor isso daqui para a frente.
Mas o problema é que o
Oi, Ralph:
Onde está o erro da solução apresentada no vídeo abaixo?
https://www.youtube.com/watch?v=c-t_BAMASKE&feature=youtu.be
Eu entendo que se um dado desarranjo tiver 2 ou mais ciclos, então quando
cada ciclo até o penúltimo for "exaurido", uma nova pessoa deverá ser
sorteada (dentre aquelas
Deixa eu copiar o que escrevi em outro lugar... :D :D
Primeiro: não fica claro do enunciado se "auto-sorteios" (alguém sortear o
próprio nome) são permitidos ou não, e isto ALTERA a resposta. :(
Vejamos possíveis respostas corretas:
---///---
SE AUTO-SORTEIOS FOREM PERMITIDOS:
Em resumo, temos
-feira, 26 de janeiro de 2021 15:22
Para: obm-l@mat.puc-rio.br
Assunto: Re: [obm-l] Amigo secreto ENEM
Olá a todos!
Vanderlei, não sou um dos especialistas da lista, mas espero que tudo bem se eu
oferecer uma humilde contribuição :-)
Creio que ainda haja outra possibilidade: considerando a
Olá a todos!
Vanderlei, não sou um dos especialistas da lista, mas espero que tudo bem
se eu oferecer uma humilde contribuição :-)
Creio que ainda haja outra possibilidade: considerando a pergunta como ela
de fato foi feita e admitindo a possibilidade de uma pessoa sortear a si
própria (o que não
Faltou mencionar que são inteiros distintos.
Em dom., 24 de jan. de 2021 às 11:35, Caio Costa
escreveu:
> Prove ou disprove que a equação acima não tem solução para x, y, a, b
> inteiros maiores que 1.
>
Em ter., 19 de jan. de 2021 às 21:25, Phablo dos Santos <
phablodosan...@gmail.com> escreveu:
> Prove que se 3<= d <= 2^(n+1), entao d nao divide [a^(2)^(n) + 1]. Para
> todo inteiro positivo a.
>
>
Seja p>2 um fator primo de a^(2^n)+1. Assim, MDC(p,a)=1 (isso deveria ser
óbvio), e portanto pelo p
Em ter., 12 de jan. de 2021 às 06:59, marcone augusto araújo borges <
marconeborge...@hotmail.com> escreveu:
> A equação ax^2 + bx + c = 0, com a, b e c inteiros tem duas raízes
> racionais cuja soma é igual ao produto. Qual a relação entre os
> coeficientes a e c?
>
As raízes são da forma p/q, p
Se aqui ninguém responder, mande um email para o...@impa.br
Abraços
Em ter., 22 de dez. de 2020 às 07:09, Maria Clara Carneiro Castro Neves <
mccneve...@gmail.com> escreveu:
> Bom dia, meu nome é Maria Clara Carneiro Castro Neves, gostaria de saber
> quando será postada a lista de convidados para
Em sáb., 5 de dez. de 2020 às 07:15, Julio César Saldaña Pumarica <
saldana...@pucp.edu.pe> escreveu:
> É verdade, 30 graus é o DAB, más a pergunta era DAC
>
> o DAC=18
>
>
> On Fri, Dec 4, 2020, 19:23 Julio César Saldaña Pumarica <
> saldana...@pucp.edu.pe> wrote:
>
>> Tenho uma solução com traç
É verdade, 30 graus é o DAB, más a pergunta era DAC
o DAC=18
On Fri, Dec 4, 2020, 19:23 Julio César Saldaña Pumarica <
saldana...@pucp.edu.pe> wrote:
> Tenho uma solução com traços auxiliares. Resposta: 30°
>
> Tem como passar uma foto nesta lista?
>
> On Mon, Nov 30, 2020, 19:42 Professor Van
Tenho uma solução com traços auxiliares. Resposta: 30°
Tem como passar uma foto nesta lista?
On Mon, Nov 30, 2020, 19:42 Professor Vanderlei Nemitz <
vanderma...@gmail.com> wrote:
> Boa noite!
> Alguém conhece uma saída para o seguinte problema?
> Muito obrigado!
>
> *Num triângulo isósceles ABC
Seja x a medida do ângulo DAC (logo DAB mede 48 -x). Por trig Ceva
sin x * sin 18 * sin 54 = sin (48-x) * sin 12 * sin 48.
Pode-se deduzir que sin 54 = (1+ sqrt(5))/4 e sin 18 = (sqrt(5)-1)/4. Logo,
sin 54 * sin 18 = 1/4. Assim, nossa equação fica
sin x / sin (48-x) = 4 * sin 12 * sin 48
usando
Não querendo polemizar, mas de acordo com o exercício, é, na minha opinião,
impossível ser 30 o ângulo pedido pq se fosse o triângulo DBC teria o lado
oposto ao ângulo de 18 menor do que o lado oposto ao ângulo de 12.
Se me enganei poderiam me mostrar, onde eu errei?
Em sex., 4 de dez. de 2020 à
Aliás, de posse da expressão para BAD e CAD, um exercício razoavelmente
fácil de programação (até em planilha), é descobrir para quais triângulos
isósceles com ângulos inteiros (em graus) e quais ângulos DBC e DCB
inteiros, BAD (e obviamente CAD) também são inteiros.
Daí, um problema (não mais um
Usando áreas - em particular, área(ABC) = (1/2)*AB*AC*sen(A) - você
consegue, com alguma facilidade, expressar a tangente de DAC em termos de
senos e cossenos dos ângulos dados. Daí, é só calcular (com calculadora
ou computador - eu uso Excel ou Wolfram Alpha). E, de fato, AD divide BAC,
que med
Use a lei dos senos e o fato de que sen(54º)-sen(18º)=sen(30º).
Em 04/12/2020 1:50, Anderson Torres escreveu:
> Em seg., 30 de nov. de 2020 às 19:28, Professor Vanderlei Nemitz
> escreveu:
>
>> Boa noite!
>>
>> Alguém conhece uma saída para o seguinte problema?
>> Muito obrigado!
>>
ou 18!?
Em sex., 4 de dez. de 2020 às 02:08, Anderson Torres <
torres.anderson...@gmail.com> escreveu:
>
>
> Em seg., 30 de nov. de 2020 às 19:28, Professor Vanderlei Nemitz <
> vanderma...@gmail.com> escreveu:
>
>> Boa noite!
>> Alguém conhece uma saída para o seguinte problema?
>> Muito obrigad
Em seg., 30 de nov. de 2020 às 19:28, Professor Vanderlei Nemitz <
vanderma...@gmail.com> escreveu:
> Boa noite!
> Alguém conhece uma saída para o seguinte problema?
> Muito obrigado!
>
> *Num triângulo isósceles ABC, AB = AC.*
> *Seja D um ponto interno tal que os ângulos DBC, DCB, DBA e DCA mede
.
Para resolver esse tipo de equações em x e y tenho agora
um modelo. Muito bom, obrigado.
Abraços,
Luís
Data: 17/11/2020
De: Claudio Buffara <claudio.buff...@gmail.com>
Para: obm-l@mat.puc-rio.br
Assunto: Re: [obm-l] Eliminar parâmetro t Você quer eliminar t em algo como:
x = at +
Você quer eliminar t em algo como:
x = at + b/t
y = ct + d/t
Pra começar, faça u = x/b e v = y/d.
Daí vem:
u = pt + 1/t
v = qt + 1/t
Isso é um sistema linear nas variáveis t e 1/t, cuja solução é:
t = (u-v)/(p-q)
1/t = (qu-pv)/(q-p)
Multiplicando as duas equações acima e eliminando denominadores
Sugestão: proponha pra eles o problema de determinar se é possível atribuir
sinais "+" ou "-" a cada um dos números:
1 2 3 4 5 6 7 8 9 10
de modo que a soma algébrica (com sinal) destes números seja igual a zero.
Isso é um desafio e é razoavelmente lúdico, apesar de envolver conceitos
que
Desculpe é q eu queria propor algo q fosse lúdico, mais um desafio,
voltada para jovens adolescentes, algo descompromissado, sem muitas
complicações com formalidades
Em qui, 12 de nov de 2020 09:10, Anderson Torres <
torres.anderson...@gmail.com> escreveu:
>
>
> Em sáb., 7 de nov. de 2020 às 16:4
Em sáb., 7 de nov. de 2020 às 16:44, Israel Meireles Chrisostomo <
israelmchrisost...@gmail.com> escreveu:
> o objetivo dessa proposta é recriar o ambiente vivido por Euler na
> época.
>
E naquele tempo eles não usavam indução? Formalização é algo bem recente na
matemática.
Sua exigência me pa
Não consigo ver nada
Em qua., 11 de nov. de 2020 às 14:52, Pedro Lazéra
escreveu:
>
Caro Romel,
Um livro que tem feito muito sucesso recente é o do Evan Chen:
https://web.evanchen.cc/geombook.html
Abraços
Samuel
Em dom., 25 de out. de 2020 às 13:19, RF escreveu:
> Bom dia!!
>
> 1- Quais os livros de Geometria indicados para preparacao para OBM e IMO?
>
> 2- Alguem tem listas
o objetivo dessa proposta é recriar o ambiente vivido por Euler na
época.
Em sáb., 7 de nov. de 2020 às 15:10, Israel Meireles Chrisostomo <
israelmchrisost...@gmail.com> escreveu:
> Na verdade eu estava elaborando um problema que dependia disso.O problema
> é esse aqui:
>
> Desafio do ano: res
Na verdade eu estava elaborando um problema que dependia disso.O problema é
esse aqui:
Desafio do ano: resolver o problema da Basiléia sem usar derivadas,
integrais, série de potências, produto infinito do seno ou cosseno, ou
mesmo indução ou números complexos.
Em sáb., 7 de nov. de 2020 às 15:07
Na verdade eu estava elaborando um problema que dependia disso.O problema é
esse aqui:
Desafio do ano: resolver o problema da Basiléia sem usar derivadas,
integrais, série de potências, produto infinito do seno ou cosseno, ou
mesmo indução.
Em sáb., 7 de nov. de 2020 às 14:47, Israel Meireles Chr
conheço uma que usa o teorema de d'lambert
Em sáb., 7 de nov. de 2020 às 12:50, Bernardo Freitas Paulo da Costa <
bernardo...@gmail.com> escreveu:
> On Thu, Nov 5, 2020 at 9:26 PM Artur Costa Steiner
> wrote:
> >
> > Para facilitar, suponhamos que o polinômio de grau n P seja mônico.
> Sejam z_1
On Thu, Nov 5, 2020 at 9:26 PM Artur Costa Steiner
wrote:
>
> Para facilitar, suponhamos que o polinômio de grau n P seja mônico. Sejam
> z_1, , z_n suas n raízes não necessariamente distintas. Para todo
> complexo z, temos que
>
> P(z) = ( z - z_1) (z - z_n)
>
> Desenvolvendo e aplican
Para facilitar, suponhamos que o polinômio de grau n P seja mônico. Sejam
z_1, , z_n suas n raízes não necessariamente distintas. Para todo
complexo z, temos que
P(z) = ( z - z_1) (z - z_n)
Desenvolvendo e aplicando o chamado produto de Stevin, vc tem as relações
de Girard.
Se o polinôm
Em qua., 28 de out. de 2020 às 08:03, Artur Costa Steiner <
artur.costa.stei...@gmail.com> escreveu:
> Achei essa prova bem imaginativa.
>
Eu acho que provar que log(n)/n tende a 0 quando n tende a infinito é
conceitualmente mais interessante.
Ou que e^n/n tende a infinito.
>
> Para n>= 2, te
Muito linda Artur.
Carlos Victor
Em 28/10/2020 7:44, Artur Costa Steiner escreveu:
> Achei essa prova bem imaginativa.
>
> Para n>= 2, temos n^(1/n) > 1. n^(1/n) pode ser escrito como
>
> n^(1/n) = ((raiz(n) . raiz(n) . 1 1)^(1/n)
>
> onde o 1 aparece n - 2 vezes. Logo, n^(1/n)
Em ter., 27 de out. de 2020 às 20:50, joao pedro b menezes <
joaopedrobmene...@gmail.com> escreveu:
> Olá, eu estava fazendo esse exercício :
> " . (OBM 2005) Dados os inteiros positivos a, c e o inteiro b, prove que
> existe um inteiro positivo x tal que a^x + x ≡ b (mod c)."
>
> Eu pensei nessa
Lembrei de outroUgo Amaldi - Elements di Geometrie.Tenho bastante livros em
pdf, me chame no pv ai conversamos.
RegisEm segunda-feira, 26 de outubro de 2020 20:44:19 BRT, RF
escreveu:
Muito obrigado por sua resposta. Voce foi o unico que deu uma ajuda :)
On 10/25/20 11:52 AM, joao pe
Eu compilei umas listas faz um bom tempo no Bitbucket. Pretendo mudar o
repositório no futuro, mas até lá divirta-se:
https://bitbucket.org/anderson_torres/junkyard/src/master/
Em seg., 26 de out. de 2020 às 20:48, Jones Colombo
escreveu:
> Oi RF -romelsfmath, um lugar para você aprender um por
Oi RF -romelsfmath, um lugar para você aprender um porção de coisas é olhar
os arquivos desta lista de problemas
http://www.mat.puc-rio.br/~obmlistas/obm-l.html lá você vai encontrar muito
material para estudar.
[@]
Jones
On Sun, Oct 25, 2020 at 1:08 PM joao pedro b menezes <
joaopedrobmene...@gma
Muito obrigado por sua resposta. Voce foi o unico que deu uma ajuda :)
On 10/25/20 11:52 AM, joao pedro b menezes wrote:
Olá, boa tarde. Eu não conheço todos, mas eu sei que é possivel entrar
no site da OBM :
https://www.obm.org.br/2020/07/25/conheca-livros-para-iniciar-a-preparacao-para-a-proxi
Correção: fazendo y=1/(r+i).
Em seg, 26 de out de 2020 às 10:49, Marcos Martinelli <
mffmartine...@gmail.com> escreveu:
> Sendo i a unidade imaginária:
>
> 1/(1+r_k^2) = 1/(2i)*(1/(r_k-i)-1/(r_k+i) (k=[1,n], onde r_k <> {-i,i}).
>
> i) Seja z_k = 1/(r_k-i) e fazendo z=1/(r-i) em r^20-7r^3+1=0:
>
Euclides - Os elementos de Geometria - Ed UnespEm domingo, 25 de outubro de
2020 13:48:59 BRT, RF escreveu:
Bom dia!!
1- Quais os livros de Geometria indicados para preparacao para OBM e IMO?
2- Alguem tem listas de Geometria preparatoria para OBM ou IMO?
Obrigado a todos
=
Sendo i a unidade imaginária:
1/(1+r_k^2) = 1/(2i)*(1/(r_k-i)-1/(r_k+i) (k=[1,n], onde r_k <> {-i,i}).
i) Seja z_k = 1/(r_k-i) e fazendo z=1/(r-i) em r^20-7r^3+1=0:
(1/z+i)^20-7(1/z+i)^3+1=0 => (1+iz)^20-7z^17(1+iz)^3+z^20=0 => (7i+2)z^20 +
(-20i+21)z^19 +...=0.
Portanto Soma_(k=[1,n]) 1/(r_k-i
De nada mano.
Em seg, 26 de out de 2020 09:40, joao pedro b menezes <
joaopedrobmene...@gmail.com> escreveu:
> Muito obrigado pela ajuda! Entendi o exercício agora.
>
> Em dom, 25 de out de 2020 às 19:59, Otávio Araújo <
> otavio17.ara...@gmail.com> escreveu:
>
>> Vc resolve essa questão mostrand
Muito obrigado pela ajuda! Entendi o exercício agora.
Em dom, 25 de out de 2020 às 19:59, Otávio Araújo
escreveu:
> Vc resolve essa questão mostrando q p=n^2+n+1. Se n=1 acabou. Se n>1,Já
> que p divide n^3-1 e é primo, temos que p divide n-1 ou n^2+n+1. Não
> podemos ter p dividindo n-1 pois n
Vc resolve essa questão mostrando q p=n^2+n+1. Se n=1 acabou. Se n>1,Já
que p divide n^3-1 e é primo, temos que p divide n-1 ou n^2+n+1. Não
podemos ter p dividindo n-1 pois n divide p-1 -> n<= p-1 n-1 p>= n+1 e k será congruente a 1 módulo n também. Suponha que k>1, k>1
implica k>= n+1 daí kp>=
Olá, boa tarde. Eu não conheço todos, mas eu sei que é possivel entrar no
site da OBM :
https://www.obm.org.br/2020/07/25/conheca-livros-para-iniciar-a-preparacao-para-a-proxima-obm/
Ainda assim, um livro que eu particularmente acho fantástico se chama
“Challenging problems in geometry “. Ele é u
Correção:
1/(1+r_k^2) = 1/(2i)*(1/(r_k-i)-1/(r_k+i)
Em dom, 25 de out de 2020 às 10:25, Marcos Martinelli <
mffmartine...@gmail.com> escreveu:
> Sendo i o complexo imaginário:
>
> 1/(1+r_k^2) = 1/(2i)*(1/(r_k-i)+1/(r_k+i)
>
> Depois você deve considerar dois novos polinômios com as seguintes
> m
Sendo i o complexo imaginário:
1/(1+r_k^2) = 1/(2i)*(1/(r_k-i)+1/(r_k+i)
Depois você deve considerar dois novos polinômios com as seguintes mudanças
de variáveis:
. x=1/y-i
. x=1/y+i
Devemos então calcular as somas dos inversos das raizes nesses dois
polinômios para termos como calcular o somat
Boa tarde!
Na verdade: 2^a=64; a= 6 e y=12.
Em qui., 22 de out. de 2020 às 11:17, Pedro José
escreveu:
> Bom dia!
> Recebi esse problema hoje: 615 + x^2 = 2^y., para x,y inteiros Não saberia
> fazer, como não soube resolver esse, acima. Mas devido a solução do colega
> Esdras, pensei:"já vi algo
Bom dia!
Recebi esse problema hoje: 615 + x^2 = 2^y., para x,y inteiros Não saberia
fazer, como não soube resolver esse, acima. Mas devido a solução do colega
Esdras, pensei:"já vi algo parecido".
Basta restringir y aos pares.
Se y é ímpar x^2=2 mod3, absurdo então y é par. Logo y=2a, com a inteiro
O algoritmo de animação não está exatamente disponível, mas o artigo da OBM
sobre o Porisma de Steiner explica bem a sua ideia: invertendo um par de
círculos concêntricos, é possível produzir qualquer configuração de Steiner.
Em sáb., 17 de out. de 2020 às 15:41, Leonardo Borges Avelino <
lbor...@
Trata-se do tema de inversão e tem um artigo na Eureka 4
https://www.obm.org.br/content/uploads/2017/01/eureka4.pdf
Abs
On Sat, Oct 17, 2020 at 3:14 PM Felippe Coulbert Balbi <
felippeba...@hotmail.com> wrote:
> A muitos anos atras durante um coloquio de matemática no IMPA, no grupo de
> olimpía
Suponha que a =1. Queremos que 1/b + 1/c seja inteiro. Mas se b >= 3, temos
1/b + 1/c <= 2/3. Logo, as únicas sol nesse subcaso são b=c=1 e b=c=2.
Vou admitir como verdade que a<4 pq vc provou isso.
Suponha que 1 < a < 4 e b >= 5. Daí
1/a + 1/b + 1/c <= 1/2 + 1/5 + 1/5 = 9/10
Logo, b < 5. (Pq n h
Há outros dois: (1,2,2) e (2,3,6).
On Tue, Oct 6, 2020 at 5:14 PM Marcos Duarte
wrote:
> Boa tarde!
>
> Encontre todos os números naturais a,b,c tais que a<=b<=c e a soma 1/a +
> 1/b + 1/c seja um inteiro.
>
> O único limitante que encontrei é que a < 4, pois 1/4 + 1/4 + 1/4 = 3/4 <
> 1 e já qu
Em sáb., 12 de set. de 2020 às 01:18, Pedro José escreveu:
>
> Boa noite!
> Atrapalhou meu vinho e o filme que estava assistindo mas consegui. Não gostei
> tanto, agora que consegui, é muito trabalhoso.
>
> 2= [3(y+1)(z+1)-1]/2yz
> yz= 3(yz+2) (i)
> z(y-3)= 3y +2 (ii)
> y(z-3)=3z+2 (iii)
> (i)*(i
Olá Luís, rabisquei aqui no papel agora, e pensei assim...
Vamos considerar primeiro o triângulo ABC inscrito no círculo, onde AB=c,
AC=b e BC=a.
Desta forma vamos considerar o problema de "ponta cabeça", onde P se
encontra no círculo e que PA=x e PC=y,
logo PC=x+y.
Vou numerar os passos para fim
Não achei uma solução na linha régua e compasso. Segue uma tentativa por
trigonometria. Dado o triângulo ABC, seja x o ângulo BAC, seja y o ângulo
ABC. Queremos P no circuncírculo tal que PB+PC=PA. Então P deve ser tal que
AP intersecta BC. Assim formamos os triângulos ABP e ACP.
Os triângulos ABC
Boa noite!
Atrapalhou meu vinho e o filme que estava assistindo mas consegui. Não
gostei tanto, agora que consegui, é muito trabalhoso.
2= [3(y+1)(z+1)-1]/2yz
yz= 3(yz+2) (i)
z(y-3)= 3y +2 (ii)
y(z-3)=3z+2 (iii)
(i)*(ii) yz(z-3)(y-3)= 9yz+6(y+z)+4 e Voilá: (z-3)(y-3)=11.
Saudações,
PJMS
Em s
Boa noite!
Fui em uma linha parecida com a primeira solução, embora não visse
necessidade de mudança de variáveis.
Mas o b achei sempre por restrição.
Esse "it implies" e aparece um número fatorado, não consegui captar, embora
tenha gostado do recurso, já que é bem restritivo.
Sudações,
PJMS
Em
Boa noite!
Grato, Ralph!
Estou estudando a solução. Pelo menos, não me decepcionei. A resposta
estava correta,
Saudações.
PJMS
Em sex., 11 de set. de 2020 às 22:33, Ralph Costa Teixeira <
ralp...@gmail.com> escreveu:
> Essa eh da IMO 1992. Tem uma solucao aqui:
> http://sms.math.nus.edu.sg/Simo
Tava dando uma olhada, vi que só com as constantes a e b não dá certo, mas
uma solução que funciona é pegar: f: (x_1, a_1y_1); (x_2,
a_2y_2);...;(x_{n+1},
a_{n+1}y_{n+1}) e g: (b_1x_1, y_1); (b_2x_2, y_2);...;(x_{n+1},
b_{n+1}y_{n+1}), com com a_i+b_i=2 e não nulos e diferentes. Daí você
mostra qu
Essa eh da IMO 1992. Tem uma solucao aqui:
http://sms.math.nus.edu.sg/Simo/IMO_Problems/92.pdf
On Fri, Sep 11, 2020 at 10:06 PM Pedro José wrote:
> Bom dia!
>
> Recebi de um filho de um amigo, um problema que já o fizera.
> (a-1)(b-1)(c-1) | abc-1; 1
> Confesso que desta feita gastei mais tempo
Acho que é assim: Dado o tal polinômio P(x), de grau n, podemos supor spdg
que P não tem raiz real (mas não é necessário) tome os pontos (x_1, y_1); (x_2,
y_2);...;(x_{n+1}, y_{n+1}) sobre o gráfico de P, onde y_i !=0. Então sejam
f e g respectivamente os polinômios de grau no máximo n que passam p
301 - 400 de 33949 matches
Mail list logo